Difference between revisions of "2016 AMC 10B Problems/Problem 16"

(Problem)
(Solution)
Line 11: Line 11:
  
 
==Solution==
 
==Solution==
 
+
The sum of an infinite geometric series is a positive number <math>S</math>, and the second term in the series is <math>1</math>. What is the smallest possible value of <math>S?</math>
 +
The sum of an infinite geometric series is of the form:
 +
<math>(a1/(1-r)</math> where <math>a1</math> is the first term and <math>r</math> is the ratio whose absolute value is less than 1.
 +
We know that the second term is the first term multiplied by the ratio.
 +
In other words:
 +
<math>a1*r=1</math>
 +
<math>a1=1/r</math>
 +
Thus the sum is the following:
 +
<math>(1/r)/(1-r)</math>
 +
We can multiply <math>r</math> to both sides of the numerator and denominator.
 +
<math>1/(r-r^2)</math>
 +
Since we want the minimum value of this expression, we want the maximum value for the denominator.
 +
<math>max(-r^2+r)</math>
 +
The maximum value of this is <math>-b/2a</math>.
 +
<math>-(1)/2(-1)=1/2</math>
 +
Plugging 1/2 in, we get:
 +
<math>1/(1/2)=2</math>, <math>B</math>
  
 
==See Also==
 
==See Also==
 
{{AMC10 box|year=2016|ab=B|num-b=15|num-a=17}}
 
{{AMC10 box|year=2016|ab=B|num-b=15|num-a=17}}
 
{{MAA Notice}}
 
{{MAA Notice}}

Revision as of 13:34, 21 February 2016

Problem

The sum of an infinite geometric series is a positive number $S$, and the second term in the series is $1$. What is the smallest possible value of $S?$

$\textbf{(A)}\ \frac{1+\sqrt{5}}{2} \qquad \textbf{(B)}\ 2 \qquad \textbf{(C)}\ \sqrt{5} \qquad \textbf{(D)}\ 3 \qquad \textbf{(E)}\ 4$


Solution

The sum of an infinite geometric series is a positive number $S$, and the second term in the series is $1$. What is the smallest possible value of $S?$ The sum of an infinite geometric series is of the form: $(a1/(1-r)$ where $a1$ is the first term and $r$ is the ratio whose absolute value is less than 1. We know that the second term is the first term multiplied by the ratio. In other words: $a1*r=1$ $a1=1/r$ Thus the sum is the following: $(1/r)/(1-r)$ We can multiply $r$ to both sides of the numerator and denominator. $1/(r-r^2)$ Since we want the minimum value of this expression, we want the maximum value for the denominator. $max(-r^2+r)$ The maximum value of this is $-b/2a$. $-(1)/2(-1)=1/2$ Plugging 1/2 in, we get: $1/(1/2)=2$, $B$

See Also

2016 AMC 10B (ProblemsAnswer KeyResources)
Preceded by
Problem 15
Followed by
Problem 17
1 2 3 4 5 6 7 8 9 10 11 12 13 14 15 16 17 18 19 20 21 22 23 24 25
All AMC 10 Problems and Solutions

The problems on this page are copyrighted by the Mathematical Association of America's American Mathematics Competitions. AMC logo.png